ΑΡΧΙΜΗΔΗΣ 2021

Συντονιστές: cretanman, ΔΗΜΗΤΡΗΣ ΙΩΑΝΝΟΥ, socrates

nikolasxen
Δημοσιεύσεις: 1
Εγγραφή: Τρί Ιουν 22, 2021 2:32 pm

Re: ΑΡΧΙΜΗΔΗΣ 2021

#61

Μη αναγνωσμένη δημοσίευση από nikolasxen » Κυρ Ιούλ 11, 2021 7:37 pm

ποτε θα δημοσιευσει η εμε τα θεματα;



Λέξεις Κλειδιά:
petrosmani
Δημοσιεύσεις: 22
Εγγραφή: Τετ Φεβ 24, 2021 6:09 pm

Re: ΑΡΧΙΜΗΔΗΣ 2021

#62

Μη αναγνωσμένη δημοσίευση από petrosmani » Τρί Ιούλ 13, 2021 5:20 pm

Τελικά θα σταλθούν τα μετάλλια σε καθένα ξεχωριστά ή θα πραγματοποιηθεί τελετή βράβευσης?


Άβαταρ μέλους
Al.Koutsouridis
Δημοσιεύσεις: 1797
Εγγραφή: Πέμ Ιαν 30, 2014 11:58 pm
Τοποθεσία: Αθήνα

Re: ΑΡΧΙΜΗΔΗΣ 2021

#63

Μη αναγνωσμένη δημοσίευση από Al.Koutsouridis » Τετ Ιούλ 21, 2021 11:09 am

Μπορεί κάποιος να ανεβάσει τα θέματα ή να αναφέρει ποια από την sortlist ήταν; Έχει περάσει η προθεσμία. Καλά αποτελέσματα στις ελληνικές εθνικές ομάδες.


DrStrange
Δημοσιεύσεις: 32
Εγγραφή: Τετ Μάιος 08, 2019 8:30 pm

Re: ΑΡΧΙΜΗΔΗΣ 2021

#64

Μη αναγνωσμένη δημοσίευση από DrStrange » Τετ Ιούλ 21, 2021 12:25 pm

Al.Koutsouridis έγραψε:
Τετ Ιούλ 21, 2021 11:09 am
Μπορεί κάποιος να ανεβάσει τα θέματα ή να αναφέρει ποια από την sortlist ήταν; Έχει περάσει η προθεσμία. Καλά αποτελέσματα στις ελληνικές εθνικές ομάδες.
Α3 G3 C2 N5


achilleas
Γενικός Συντονιστής
Δημοσιεύσεις: 3014
Εγγραφή: Τρί Σεπ 15, 2009 3:32 pm

Re: ΑΡΧΙΜΗΔΗΣ 2021

#65

Μη αναγνωσμένη δημοσίευση από achilleas » Πέμ Ιούλ 22, 2021 4:21 am

Al.Koutsouridis έγραψε:
Τετ Ιούλ 21, 2021 11:09 am
Μπορεί κάποιος να ανεβάσει τα θέματα ή να αναφέρει ποια από την sortlist ήταν; Έχει περάσει η προθεσμία. Καλά αποτελέσματα στις ελληνικές εθνικές ομάδες.
Ανεβάζω τα πρώτα δύο των μεγάλων προς το παρόν που τα έχω έτοιμα:

Πρόβλημα 1. Έστω a,b,c,d θετικοί πραγματικοί αριθμοί τέτοιοι ώστε (a+c)(b+d)=ac+bd. Να προσδιορίσετε την ελάχιστη δυνατή τιμή της παράστασης
\displaystyle  
	A=\frac{a}{b}+\frac{b}{c}+\frac{c}{d}+\frac{d}{a}.


Πρόβλημα 2. Έστω AB\Gamma\Delta κυρτό τετράπλευρο με A\widehat{B}\Gamma>90^\circ, \Gamma\widehat{\Delta}A>90^\circ και \Delta\widehat{A}B=B\widehat{\Gamma}\Delta. Έστω E και Z τα συμμετρικά του A ως προς τις ευθείες B\Gamma και \Gamma\Delta, αντίστοιχα. Υποθέτουμε ότι τα τμήματα AE και AZ τέμνουν την ευθεία B\Delta στα σημεία K και \Lambda, αντίστοιχα. Να αποδείξετε ότι οι περιγεγραμμένοι κύκλοι των τριγώνων BEK και \Delta Z\Lambda εφάπτονται μεταξύ τους.

Φιλικά,

Αχιλλέας


Joaakim
Δημοσιεύσεις: 120
Εγγραφή: Σάβ Φεβ 22, 2020 4:40 pm
Τοποθεσία: Θεσσαλονίκη

Re: ΑΡΧΙΜΗΔΗΣ 2021

#66

Μη αναγνωσμένη δημοσίευση από Joaakim » Πέμ Ιούλ 22, 2021 11:55 am

achilleas έγραψε:
Πέμ Ιούλ 22, 2021 4:21 am

Πρόβλημα 1. Έστω a,b,c,d θετικοί πραγματικοί αριθμοί τέτοιοι ώστε (a+c)(b+d)=ac+bd. Να προσδιορίσετε την ελάχιστη δυνατή τιμή της παράστασης
\displaystyle  
	A=\frac{a}{b}+\frac{b}{c}+\frac{c}{d}+\frac{d}{a}.
Από την Ανισότητα Αριθμητικού- Γεωμετρικού Μέσου (AM-GM) έχουμε ότι:

\displaystyle A=(\frac{a}{b}+\frac{c}{d})+(\frac{b}{c}+\frac{d}{a}) \geq

\displaystyle \geq 2 \sqrt {\frac{ac}{bd}}+2 \sqrt {\frac{bd}{ac}}=

\displaystyle = \frac{2(ac+bd)}{\sqrt{abcd}}=\frac{2(a+c)(b+d)}{\sqrt{abcd}} \geq

\displaystyle \geq \frac{2 \cdot 2 \sqrt{ac} \cdot 2 \sqrt{bd}}{\sqrt{abcd}}=

\displaystyle = \frac{8 \sqrt{abcd}}{\sqrt{abcd}}=8 \Rightarrow A_{min.}=8,

με την ισότητα να πιάνεται για a=c, b=d.


achilleas
Γενικός Συντονιστής
Δημοσιεύσεις: 3014
Εγγραφή: Τρί Σεπ 15, 2009 3:32 pm

Re: ΑΡΧΙΜΗΔΗΣ 2021

#67

Μη αναγνωσμένη δημοσίευση από achilleas » Πέμ Ιούλ 22, 2021 2:04 pm

achilleas έγραψε:
Πέμ Ιούλ 22, 2021 4:21 am
...
Πρόβλημα 1. Έστω a,b,c,d θετικοί πραγματικοί αριθμοί τέτοιοι ώστε (a+c)(b+d)=ac+bd. Να προσδιορίσετε την ελάχιστη δυνατή τιμή της παράστασης
\displaystyle  
	A=\frac{a}{b}+\frac{b}{c}+\frac{c}{d}+\frac{d}{a}.
Λύση: Θέτουμε x=\dfrac{a}{b} και y=\dfrac{c}{d}. Τότε a=bx και c=dy, και η δοθείσα σχέση \displaystyle (a+c)(b+d)=ac+bd δίνει
 
(bx+dy)(b+d)=(bx)(dy)+bd.

Αφού διαιρέσουμε και τα δύο μέλη με bd>0 και εφαρμόσουμε την την επιμεριστική ιδιότητα παίρνουμε
 
\frac{bx}{d}+\frac{dy}{b}+x+y=xy+1.

Από την ανισότητα ΑΜ-ΓΜ παίρνουμε
 
\frac{bx}{d}+\frac{dy}{b}\geq 2\sqrt{\frac{bx}{d}\cdot \frac{dy}{b}}=2\sqrt{xy} και  
 x+y\geq 2\sqrt{xy}.

Έτσι  
xy+1\geq 4\sqrt{xy}. 
Έχουμε
\displaystyle{ 
\begin{aligned} 
A=x+\frac{b}{c}+y+\frac{d}{a}&=(x+y)+\left(\frac{b}{c}+\frac{d}{a}\right)\\ 
	&\geq 2\sqrt{xy}+2\sqrt{\frac{bd}{ca}}\\ 
	&=2\left(\sqrt{xy}+\frac{1}{\sqrt{xy}}\right)\\ 
	&=2\cdot \frac{xy+1}{\sqrt{xy}}\\ 
	&\geq 2\cdot  \frac{4\sqrt{xy}}{\sqrt{xy}}\\ 
	&=8.\notag 
\end{aligned} 
}

Η ισότητα ισχύει εάν x=y και b=d, δηλ. εάν a=c και b=d. Τότε η δοθείσα σχέση γράφεται 4ab=a^2+b^2, που δίνει b=(2+\sqrt{3})a ή b=(2-\sqrt{3})a.

Συνεπώς, η ελάχιστη δυνατή τιμή της παράστασης είναι 8, η οποία λαμβάνεται για όλες τις τετράδες (a,b,c,d) της μορφής (t,(2+\sqrt{3})t,t,(2+\sqrt{3})t) και (t,(2-\sqrt{3})t,t,(2-\sqrt{3})t) με t>0.


petrosmani
Δημοσιεύσεις: 22
Εγγραφή: Τετ Φεβ 24, 2021 6:09 pm

Re: ΑΡΧΙΜΗΔΗΣ 2021

#68

Μη αναγνωσμένη δημοσίευση από petrosmani » Πέμ Ιούλ 22, 2021 2:23 pm

Στο 1 πρόβλημα των μικρών 8 είναι η απάντηση όπως και στους μεγάλους ?


achilleas
Γενικός Συντονιστής
Δημοσιεύσεις: 3014
Εγγραφή: Τρί Σεπ 15, 2009 3:32 pm

Re: ΑΡΧΙΜΗΔΗΣ 2021

#69

Μη αναγνωσμένη δημοσίευση από achilleas » Πέμ Ιούλ 22, 2021 2:40 pm

petrosmani έγραψε:
Πέμ Ιούλ 22, 2021 2:23 pm
Στο 1 πρόβλημα των μικρών 8 είναι η απάντηση όπως και στους μεγάλους ?
Ναι. Δείτε μια παραλλαγή εδώ.


achilleas
Γενικός Συντονιστής
Δημοσιεύσεις: 3014
Εγγραφή: Τρί Σεπ 15, 2009 3:32 pm

Re: ΑΡΧΙΜΗΔΗΣ 2021

#70

Μη αναγνωσμένη δημοσίευση από achilleas » Πέμ Ιούλ 22, 2021 2:45 pm

achilleas έγραψε:
Πέμ Ιούλ 22, 2021 4:21 am
..
Πρόβλημα 2. Έστω AB\Gamma\Delta κυρτό τετράπλευρο με A\widehat{B}\Gamma>90^\circ, \Gamma\widehat{\Delta}A>90^\circ και \Delta\widehat{A}B=B\widehat{\Gamma}\Delta. Έστω E και Z τα συμμετρικά του A ως προς τις ευθείες B\Gamma και \Gamma\Delta, αντίστοιχα. Υποθέτουμε ότι τα τμήματα AE και AZ τέμνουν την ευθεία B\Delta στα σημεία K και \Lambda, αντίστοιχα. Να αποδείξετε ότι οι περιγεγραμμένοι κύκλοι των τριγώνων BEK και \Delta Z\Lambda εφάπτονται μεταξύ τους.
..
Λύση: Έστω H το συμμετρικό του σημείου A ως προς την B\Delta. Έστω L το σημείο τομής της B\Gamma με την AE και έστω M το σημείο τομής της B\Delta με την AH. Τότε το BL είναι ύψος, διχοτόμος και διάμεσος στο \triangle EBA, και ομοίως το BM στο \triangle ABH.

Έχουμε

 
\begin{aligned}\notag 
	E\widehat{B}H&=360^\circ-E\widehat{B}A-A\widehat{B}H\\ 
				&=360^\circ-2L\widehat{B}A-2A\widehat{B}M\\ 
				&=2(180^\circ-L\widehat{B}M)\\ 
				&=2L\widehat{A}M \\ 
				&=E\widehat{K}H, 
\end{aligned}

Συνεπώς, το H ανήκει στον περιγεγραμμένο κύκλο του \triangle BEK.

Ομοίως, εάν N είναι το σημείο τομής της \Gamma\Delta και της AZ είναι

 
\begin{aligned}\notag 
	Z\widehat{\Delta}H&=360^\circ-H\widehat{\Delta}A-Z\widehat{\Delta}A\\ 
				&=360^\circ-2M\widehat{\Delta}A-2A\widehat{\Delta}N\\ 
				&=2(180^\circ-M\widehat{\Delta}N)\\\notag 
				&=2M\widehat{A}N \\ 
				&=H\widehat{\Lambda}Z. 
\end{aligned}

Άρα, το H ανήκει και στον περιγεγραμμένο κύκλο του \Delta Z\Lambda. Συνεπώς, οι περιγεγραμμένοι κύκλοι των τριγώνων BEK και \Delta Z\Lambda τέμνονται στο H. Για να δείξουμε ότι εφάπτονται σε αυτό αρκεί να δείξουμε ότι F\widehat{H}G=180^\circ, όπου F, G είναι τα αντίστοιχα κέντρα τους (βλ. σχήμα).

Πράγματι, αφού τα ισοσκελή τρίγωνα BFH και EFB είναι ίσα, έχουμε

\displaystyle  
F\widehat{H}B=\frac{E\widehat{B}H}{2}=\frac{E\widehat{K}H}{2}=K\widehat{A}H=L\widehat{A}H.

Ομοίως,

\displaystyle  
\Delta\widehat{H}G=\frac{Z\widehat{\Delta}H}{2}=\frac{H\widehat{\Lambda} Z}{2}=H\widehat{A}\Lambda=H\widehat{A}N.

Αφού \displaystyle B\widehat{H}\Delta=B\widehat{A}\Delta=B\widehat{\Gamma}\Delta=L\widehat{\Gamma}N,

έχουμε

\displaystyle  
F\widehat{H}G=F\widehat{H}B+B\widehat{H}\Delta+\Delta\widehat{H}G=L\widehat{A}H+L\widehat{\Gamma}N+H\widehat{A}N=L\widehat{A}N+L\widehat{\Gamma}N=180^\circ,

στο AL\Gamma N, όπως θέλαμε.
Συνημμένα
archimedes_2021_problem_2_forum.png
archimedes_2021_problem_2_forum.png (44.45 KiB) Προβλήθηκε 2013 φορές


Άβαταρ μέλους
Lymperis Karras
Δημοσιεύσεις: 170
Εγγραφή: Παρ Νοέμ 06, 2020 5:16 pm

Re: ΑΡΧΙΜΗΔΗΣ 2021

#71

Μη αναγνωσμένη δημοσίευση από Lymperis Karras » Πέμ Ιούλ 22, 2021 3:14 pm

achilleas έγραψε:
Πέμ Ιούλ 22, 2021 4:21 am
Πρόβλημα 2. Έστω AB\Gamma\Delta κυρτό τετράπλευρο με A\widehat{B}\Gamma>90^\circ, \Gamma\widehat{\Delta}A>90^\circ και \Delta\widehat{A}B=B\widehat{\Gamma}\Delta. Έστω E και Z τα συμμετρικά του A ως προς τις ευθείες B\Gamma και \Gamma\Delta, αντίστοιχα. Υποθέτουμε ότι τα τμήματα AE και AZ τέμνουν την ευθεία B\Delta στα σημεία K και \Lambda, αντίστοιχα. Να αποδείξετε ότι οι περιγεγραμμένοι κύκλοι των τριγώνων BEK και \Delta Z\Lambda εφάπτονται μεταξύ τους.

Φιλικά,

Αχιλλέας
Edit: Με πρόλαβε ο κ. Αχιλλέας. Η λύση ήταν σχεδόν ίδια με την δική μου.


Ένας μαθηματικός χρειάζεται μολύβι, γόμα και μεγάλο καλάθι αχρήστων.
-Hilbert
petrosmani
Δημοσιεύσεις: 22
Εγγραφή: Τετ Φεβ 24, 2021 6:09 pm

Re: ΑΡΧΙΜΗΔΗΣ 2021

#72

Μη αναγνωσμένη δημοσίευση από petrosmani » Πέμ Ιούλ 22, 2021 3:21 pm

achilleas έγραψε:
Πέμ Ιούλ 22, 2021 2:40 pm
petrosmani έγραψε:
Πέμ Ιούλ 22, 2021 2:23 pm
Στο 1 πρόβλημα των μικρών 8 είναι η απάντηση όπως και στους μεγάλους ?
Ναι. Δείτε μια παραλλαγή εδώ.
AAAAAA Ωραία και εγώ τόσο βρήκα αλλά δεν ήξερα αν ηταν σωστο γιατί με μπέρδεψε καθώς χ+1/χ>=2 και ετσι οταν το έλυσα ξανά νόμιζα οτι η απάντηση ειναι το 4. Μπορεί κάποις να μου το εξηγήσει αφου το 4 ειναι μικρότερο απο το 8.


Άβαταρ μέλους
llenny
Δημοσιεύσεις: 74
Εγγραφή: Τρί Απρ 23, 2019 11:10 pm
Τοποθεσία: Ηράκλειο Κρήτης

Re: ΑΡΧΙΜΗΔΗΣ 2021

#73

Μη αναγνωσμένη δημοσίευση από llenny » Πέμ Ιούλ 22, 2021 3:24 pm

petrosmani έγραψε:
Πέμ Ιούλ 22, 2021 3:21 pm
achilleas έγραψε:
Πέμ Ιούλ 22, 2021 2:40 pm
petrosmani έγραψε:
Πέμ Ιούλ 22, 2021 2:23 pm
Στο 1 πρόβλημα των μικρών 8 είναι η απάντηση όπως και στους μεγάλους ?
Ναι. Δείτε μια παραλλαγή εδώ.
AAAAAA Ωραία και εγώ τόσο βρήκα αλλά δεν ήξερα αν ηταν σωστο γιατί με μπέρδεψε καθώς χ+1/χ>=2 και ετσι οταν το έλυσα ξανά νόμιζα οτι η απάντηση ειναι το 4. Μπορεί κάποις να μου το εξηγήσει αφου το 4 ειναι μικρότερο απο το 8.
Διότι για να επιτύχουμε το 4 πρέπει όλες οι μεταβλητές να είναι ίσες και τότε δεν ικανοποιείται η συνθήκη που μας δίνεται πάνω.
τελευταία επεξεργασία από llenny σε Πέμ Ιούλ 22, 2021 3:25 pm, έχει επεξεργασθεί 1 φορά συνολικά.


petrosmani
Δημοσιεύσεις: 22
Εγγραφή: Τετ Φεβ 24, 2021 6:09 pm

Re: ΑΡΧΙΜΗΔΗΣ 2021

#74

Μη αναγνωσμένη δημοσίευση από petrosmani » Πέμ Ιούλ 22, 2021 3:25 pm

Ευχαριστώ.


Άβαταρ μέλους
silouan
Επιμελητής
Δημοσιεύσεις: 1398
Εγγραφή: Τρί Ιαν 27, 2009 10:52 pm

Re: ΑΡΧΙΜΗΔΗΣ 2021

#75

Μη αναγνωσμένη δημοσίευση από silouan » Πέμ Ιούλ 22, 2021 5:17 pm

Πρόβλημα 3. Έστω k ένας θετικός ακέραιος. Οι κορυφές ενός κυρτού (4k+1)-γωνου P χρωματίζονται με δύο χρώματα, άσπρο και μαύρο και κάθε χρώμα χρησιμοποιείται τουλάχιστον k φορές. Ένα κυρτό τετράπλευρο, που οι κορυφές του είναι και κορυφές του P, θα ονομάζεται καλό, εάν έχει τρεις κορυφές του ενός χρώματος και μία κορυφή του άλλου χρώματος. Να αποδείξετε ότι υπάρχουν k καλά κυρτά τετράπλευρα που είναι ανά δύο ξένα, δηλαδή ανά δύο δεν έχουν κοινά σημεία στην περίμετρο ή στο εσωτερικό τους.


Σιλουανός Μπραζιτίκος
Άβαταρ μέλους
Nick1990
Δημοσιεύσεις: 669
Εγγραφή: Παρ Ιαν 23, 2009 3:15 pm
Τοποθεσία: Peking University, Πεκίνο

Re: ΑΡΧΙΜΗΔΗΣ 2021

#76

Μη αναγνωσμένη δημοσίευση από Nick1990 » Πέμ Ιούλ 22, 2021 8:16 pm

silouan έγραψε:
Πέμ Ιούλ 22, 2021 5:17 pm
Πρόβλημα 3. Έστω k ένας θετικός ακέραιος. Οι κορυφές ενός κυρτού (4k+1)-γωνου P χρωματίζονται με δύο χρώματα, άσπρο και μαύρο και κάθε χρώμα χρησιμοποιείται τουλάχιστον k φορές. Ένα κυρτό τετράπλευρο, που οι κορυφές του είναι και κορυφές του P, θα ονομάζεται καλό, εάν έχει τρεις κορυφές του ενός χρώματος και μία κορυφή του άλλου χρώματος. Να αποδείξετε ότι υπάρχουν k καλά κυρτά τετράπλευρα που είναι ανά δύο ξένα, δηλαδή ανά δύο δεν έχουν κοινά σημεία στην περίμετρο ή στο εσωτερικό τους.
Θα χρησιμοποιήσουμε επαγωγή. Αρχικά για k = 1 έχουμε ένα πεντάγωνο ABCDE με τουλάχιστον 1 άσπρη και τουλάχιστον 1 μαύρη κορυφή. Αρκεί να εξετάσουμε τις περιπτώσεις 1 άσπρης και 4 μαύρων κορυφών, και 2 άσπρων και 3 μαύρων κορυφών. Στην πρώτη περίπτωση υπάρχει καλό τετράπλευρο με τη μια άσπρη κορυφή και 3 από τις 4 μαύρες, ενώ στη δεύτερη υπάρχει καλό τετράπλευρο με 1 από τις 2 άσπρες κορυφές και τις 3 μαύρες.

Πάμε τώρα στο επαγωγικό βήμα. Έστω x και y το πλήθος των άσπρων και των μαύρων κορυφών αντίστοιχα. Χωρίς βλάβη της γενικότητας, έστω x \geq y. Τότε έχουμε y \geq k και x \geq \frac{x + y}{2} = 2k + \frac{1}{2} \Rightarrow x \geq 2k + 1. Θα δείξουμε πως υπάρχουν 4 διαδοχικές κορυφές που σχηματίζουν καλό τετράπλευρο με 3 άσπρες κορυφές. Αν P = A_1A_2...A_{4k+1}, για κάθε m \leq 4k + 1 ορίζουμε a_m = 1 αν η A_m είναι άσπρη και a_m = 0 διαφορετικά, και στη συνέχεια ορίζουμε S_m = a_m + a_{m + 1 (mod(4k+1))} + a_{m + 2 (mod(4k+1))} + a_{m + 3 (mod(4k+1))}, οπότε το ζητούμενο σύνολο 4 διαδοχικών κορυφών σχηματίζεται αν S_m = 3 για κάποιο m. Παρατηρούμε ότι τα S_m και S_{m+1} διαφέρουν πάντα το πολύ κατά 1, ενώ S_m \in \{0, 1, 2, 3, 4\} για κάθε m, οπότε ο μόνος τρόπος να μην υπάρχει m με S_m = 3 είναι να έχουμε S_m = 4 για όλα τα m, ή να έχουμε S_m \in \{0, 1, 2\} για κάθε m. Η πρώτη περίπτωση δίνει εύκολα ότι όλες οι κορυφές είναι άσπρες, το οποίο αντιβαίνει στις υποθέσεις του προβλήματος. Στη δεύτερη περίπτωση πάλι έχουμε:

\displaystyle{\sum_{m = 1}^{4k+1}{S_m} = 4\sum_{m = 1}^{4k+1}{a_m} = 4x \geq 4(2k + 1) = 8k + 4}

και ταυτόχρονα

\displaystyle{\sum_{m = 1}^{4k+1}{S_m} \leq (4k + 1)\times 2 = 8k + 2}

άτοπο και πάλι. Επομένως, υπάρχει m με S_m = 3, άρα και 4 διαδοχικές κορυφές που σχηματίζουν καλό τετράπλευρο Τ με 3 άσπρες κορυφές. Σβήνοντας αυτό το καλό τετράπλευρο (τις κορυφές του και κάθε ακμή που διέρχεται από κάποια εξ' αυτών), μένει ένα κυρτό πολύγωνο P' που ικανοποιεί τις υποθέσεις του προβλήματος για k - 1, αφού έχει 4(k - 1) + 1 κορυφές, τουλάχιστον k - 1 μαύρες κορυφές και τουλάχιστον 2k + 1 - 3 = 2(k - 1) \geq k - 1 άσπρες κορυφές. Από την επαγωγική υπόθεση, το P' έχει k - 1 ξένα ανά 2 καλά κυρτά τετράπλευρα, τα οποία δεν τέμνουν το T (αφού το P' δεν τέμνει το T), οπότε μαζί με το T έχουμε k ξένα ανά 2 καλά κυρτά τετράπλευρα.


Κολλιοπουλος Νικος.
Μεταδιδακτορικός ερευνητής.
Ερευνητικά ενδιαφέροντα: Στοχαστικές ΜΔΕ, ασυμπτωτική ανάλυση στοχαστικών συστημάτων, εφαρμογές αυτών στα χρηματοοικονομικά και στη διαχείριση ρίσκων.
2nisic
Δημοσιεύσεις: 220
Εγγραφή: Παρ Δεκ 04, 2020 12:06 pm

Re: ΑΡΧΙΜΗΔΗΣ 2021

#77

Μη αναγνωσμένη δημοσίευση από 2nisic » Πέμ Ιούλ 22, 2021 8:23 pm

silouan έγραψε:
Πέμ Ιούλ 22, 2021 5:17 pm
Πρόβλημα 3. Έστω k ένας θετικός ακέραιος. Οι κορυφές ενός κυρτού (4k+1)-γωνου P χρωματίζονται με δύο χρώματα, άσπρο και μαύρο και κάθε χρώμα χρησιμοποιείται τουλάχιστον k φορές. Ένα κυρτό τετράπλευρο, που οι κορυφές του είναι και κορυφές του P, θα ονομάζεται καλό, εάν έχει τρεις κορυφές του ενός χρώματος και μία κορυφή του άλλου χρώματος. Να αποδείξετε ότι υπάρχουν k καλά κυρτά τετράπλευρα που είναι ανά δύο ξένα, δηλαδή ανά δύο δεν έχουν κοινά σημεία στην περίμετρο ή στο εσωτερικό τους.
Για k=1 προφανώς και ισχύει από συνθήκη και από την αρχή της περιστεροφωλιας.

Έστω ότι ισχύει για k=n θα δείξουμε ότι ισχύει και για k=n+1.
(Σκοπός μας είναι να δείξουμε πως υπάρχουν 4 διαδοχικές κορυφές οι οποίες αποτελούν καλό τετράπλευρο τότε από το επαγωγικό βήμα έχουμε τελείωση).

Από την αρχή τής περιστεροφωλιας έχουμε ότι ένα χρώμα έχει τουλάχιστον 2n+3 κορυφές έστω μαύρες.
Επιλέγω μια λευκή κορυφή και χωρίς αυτή φτιάχνω n+1 ομάδες διαδοχικών τετράδων κορυφών.

Από την αρχή της περιστεροφωλιας και πάλη έχουμε ότι μια τετράδα περιέχει τουλάχιστον τρεις μαύρες κοριφες.
Αν περιέχει τρεις μαύρες και μια λευκή τελειώσαμε.

Αν και τα 4 έχουν το ίδιο χρώμα τότε κάνουμε ένα "κλικ" δεξιά όπως η φορά του ρολογιού αν είναι 3-1 τελειώσαμε αν είναι πάλη όλα το ίδιο χρώμα το ξανακάνουμε μέχρι να καταλήξουμε σε 3-1 που είναι σίγουρο αφού υπάρχουν και από τα δύο χρώματα.


petrosmani
Δημοσιεύσεις: 22
Εγγραφή: Τετ Φεβ 24, 2021 6:09 pm

Re: ΑΡΧΙΜΗΔΗΣ 2021

#78

Μη αναγνωσμένη δημοσίευση από petrosmani » Παρ Ιούλ 23, 2021 12:03 pm

Ξέρει κάποιος αν θα βγουν οι βαθμολογίες ?


achilleas
Γενικός Συντονιστής
Δημοσιεύσεις: 3014
Εγγραφή: Τρί Σεπ 15, 2009 3:32 pm

Re: ΑΡΧΙΜΗΔΗΣ 2021

#79

Μη αναγνωσμένη δημοσίευση από achilleas » Παρ Ιούλ 23, 2021 9:16 pm

Πρόβλημα 4. Έστω \mathbb{N} το σύνολο των φυσικών αριθμών και \mathbb{N}^*=\mathbb{N}-\{0\} το σύνολο των μη μηδενικών φυσικών αριθμών. Να προσδιορίσετε όλες τις συναρτήσεις f:\mathbb{N}^*\to \mathbb{N} οι οποίες ικανοποιούν και τις τρεις συνθήκες:

(α) f(n)\ne 0 για ένα τουλάχιστον n\in \mathbb{N}^*,
(β) f(xy)=f(x){\color{red}{+}}f(y) για κάθε x,y\in \mathbb{N}^*,
(γ) υπάρχουν άπειροι θετικοί φυσικοί αριθμοί n έτσι ώστε: f(k)=f(n-k), για όλα τα k με 0<k<n.


Άβαταρ μέλους
Nick1990
Δημοσιεύσεις: 669
Εγγραφή: Παρ Ιαν 23, 2009 3:15 pm
Τοποθεσία: Peking University, Πεκίνο

Re: ΑΡΧΙΜΗΔΗΣ 2021

#80

Μη αναγνωσμένη δημοσίευση από Nick1990 » Παρ Ιούλ 23, 2021 11:48 pm

achilleas έγραψε:
Παρ Ιούλ 23, 2021 9:16 pm
Πρόβλημα 4. Έστω \mathbb{N} το σύνολο των φυσικών αριθμών και \mathbb{N}^*=\mathbb{N}-\{0\} το σύνολο των μη μηδενικών φυσικών αριθμών. Να προσδιορίσετε όλες τις συναρτήσεις f:\mathbb{N}^*\to \mathbb{N} οι οποίες ικανοποιούν και τις τρεις συνθήκες:

(α) f(n)\ne 0 για ένα τουλάχιστον n\in \mathbb{N}^*,
(β) f(xy)=f(x)f(y) για κάθε x,y\in \mathbb{N}^*,
(γ) υπάρχουν άπειροι θετικοί φυσικοί αριθμοί n έτσι ώστε: f(k)=f(n-k), για όλα τα k με 0<k<n.
Απ' όσο θυμάμαι, η δεύτερη συνθήκη είναι f(xy) = f(x) + f(y) για κάθε x, y

Αρχικά από το β) έχουμε f(1) = f(1) + f(1) \Rightarrow f(1) = 0, άρα n > 1.

Υποθέτουμε τώρα (χωρίς βλάβη της γενικότητας) ότι n είναι ο μικρότερος φυσικός τέτοιος ώστε f(n) \neq 0. Στη συνέχεια, αν n = p_1^{a_1}...p_k^{a_k} είναι η παραγοντοποίηση του n σε δυνάμεις πρώτων, από το β) έχουμε f(n) = a_1f(p_1) + ... + a_kf(p_k) \neq 0, που σημαίνει ότι f(p_j) \neq 0 για κάποιο j. Ισχύει όμως p_j \leq n, οπότε από την υπόθεση ελαχίστου πρέπει να έχουμε n = p_j, που σημαίνει ότι n = p όπου p είναι ένας πρώτος.

Θα δείξουμε τώρα ότι f(n) = kv_p(n) για κάθε n, όπου k ένας θετικός ακέραιος και v_p(n) είναι η μέγιστη δύναμη του p που διαιρεί το n.

Έστω τώρα n ένας φυσικός με f(k)=f(n-k), για όλα τα k με 0<k<n. Αν n \geq p, διαιρώντας το n με το p έχουμε n = dp + v με 0 \leq v < p. Μετά, αν είναι v > 0 τότε έχουμε f(v) = f(n - v) = f(dp) = f(d) + f(p) \geq f(p) > 0, το οποίο είναι άτοπο διότι v < p και ο p είναι ο ελάχιστος n με f(n) \neq 0. Άρα είναι v = 0 και n = dp. Μετά για κάθε 0 < k < d έχουμε 0 < kp < dp = n και άρα f((d - k)p) = f(n - kp) = f(kp) \Leftrightarrow f(p) + f(d - k) = f(k) + f(p) δηλαδή f(d - k) = f(k). Άρα ο d έχει την ίδια ιδιότητα με τον n, οπότε επαναλαμβάνοντας την ίδια διαδικασία βρίσκουμε μια γνησίως φθίνουσα ακολουθία d_1 = n = pd > d = d_2 = pd_3 > d_3 = pd_4 > d_4 > ... > d_{m - 1} = pd_m > d_m με d_m < p, όπου κάθε d_i έχει την ίδια ιδιότητα με τον n (η κατασκευή της ακολουθίας μπορεί να συνεχιστεί μέχρι να φτάσουμε σε όρο μικρότερο του p, και αυτό πρέπει να συμβεί μετά από πεπερασμένο αριθμό βημάτων καθώς διαφορετικά θα είχαμε μια άπειρη γνησίως φθίνουσα ακολουθία φυσικών αριθμών, πράγμα αδύνατο). Επομένως για q = d_m παίρνουμε n = qp^{m-1}, όπου q < p και m \in \mathbb{N}^{*}. Επιπλέον, επειδή υπάρχουν άπειροι φυσικοί n με την ιδιότητα γ) και q \in \{1, 2, ..., p - 1\}, υπάρχει ένας q ώστε τη συγκεκριμένη ιδιότητα να την έχουν άπειροι αριθμοί της μορφής qp^{m-1} με m \in \mathbb{N}^{*}, οπότε τελικά όλοι οι αριθμοί αυτής της μορφής (από την κατασκευή της γνησίως φθίνουσας ακολουθίας παραπάνω). Τέλος, επειδή για 0 < k' < p^{m-1} με m > 1 έχουμε 0 < qk' < qp^{m-1}, ισχύει f(qk') = f(qp^{m-1} - qk') = f(q(p^{m-1} - k')), οπότε από το β) έχουμε f(q) + f(k') = f(q) + f(p^{m-1} - k') \Leftrightarrow f(k') = f(p^{m-1} - k'), δηλαδή την ιδιότητα γ) την έχουν όλες οι δυνάμεις το p με θετικό ακέραιο εκθέτη.

Θέτουμε τώρα k = f(p), και για έναν οποιοδήποτε φυσικό n έχουμε n = p^{v_p(n)}q με v_p(q) = 0, οπότε από το β) έχουμε f(n) = f(p^{v_p(n)}) + f(q) = f(p)v_p(n) + f(q) = kv_p(n) + f(q), οπότε αρκεί να δείξουμε ότι f(q) = 0. Έστω πως υπάρχουν φυσικοί r που δεν διαιρούνται με το p ώστε f(r) > 0, και έστω r_0 ο ελάχιστος αυτών. Τότε, υπάρχει φυσικός m > 1 με p^m < r_0 < p^{m+1}, οπότε γράφουμε p^{m+1} = dr_0 + v με 0 \leq v < r_0 και έχουμε f(v) = f(p^{m+1} - v) = f(dr_0) = f(d) + f(r_0) \geq f(r_0) > 0, που σημαίνει ότι v = pv_0 (από την υπόθεση ελαχίστου για το r_0). Άρα έχουμε p^{m+1} = dr_0 + pv_0 με gcd(r_0, p) = 1, οπότε πρέπει d = pd_0 και p^{m+1} = pd_0r_0 + pv_0 \Rightarrow p^m = d_0r_0 + v_0 \geq r_0 αν d_0 > 0 που είναι άτοπο αφού p^m < r_0 < p^{m+1}. Επομένως d_0 = 0 και d = pd_0 = 0, που σημαίνει ότι p^{m+1} = v < r_0 που είναι πάλι άτοπο αφού p^m < r_0 < p^{m+1}. Άρα έχουμε πράγματι f(q) = 0 οποτεδήποτε ο q δεν διαιρείται με τον p και άρα f(n) = kv_p(n) για κάθε φυσικό n.

Εύκολα όλες οι συναρτήσεις της παραπάνω μορφής ικανοποιούν τη συνθήκη α) για n = p, τη συνθήκη β), και τη συνθήκη γ) για τις δυνάμεις του p.


Κολλιοπουλος Νικος.
Μεταδιδακτορικός ερευνητής.
Ερευνητικά ενδιαφέροντα: Στοχαστικές ΜΔΕ, ασυμπτωτική ανάλυση στοχαστικών συστημάτων, εφαρμογές αυτών στα χρηματοοικονομικά και στη διαχείριση ρίσκων.
Απάντηση

Επιστροφή σε “Θέματα διαγωνισμών (ΕΜΕ, ΚΥΜΕ, BMO, JBMO, IMO, Kangaroo κλπ)”

Μέλη σε σύνδεση

Μέλη σε αυτήν τη Δ. Συζήτηση: Δεν υπάρχουν εγγεγραμμένα μέλη και 4 επισκέπτες